0 Daumen
689 Aufrufe

meine erste Frage, also entschuldige ich mich für das Formatieren


Aufgabe:

Gegeben sei f(x,y,z) = x und die zwei Nebenbegingungen F(x,y,z) = x^2 + y^2 + z^2 -1 und G(x,y,z) = x^3 + y^3 + z^3.

F = G = 0 und ich muss nun das Maximum der Funktion f finden.


Problem/Ansatz:

Mein Problem ist, dass ich zuerst versucht habe mit der Determinanten Version zu lösen und komme nicht auf das richtige Resultat.

Meine Matrix lautet doch

12x\( 3x^{2} \)
02y\( 3y^{2} \)
02z\( 3z^{2} \)

und somit aus der Determinanten z = y. Dies sollte ich doch in die NB einfügen können und danach mein Maximum gefunden haben? Also;

\( x^{2} \) + \( 2z^{2} \) = 1

\( x^{3} \) + \( 2z^{3} \) = 0

Dies für x und z aufgelöst ergibt dann -0.665 = x und 0.5279 = z und das selbe mit anderen Vorzeichen


Meine Musterlösung löst die Aufgabe anders und kommt auch nicht auf das selbe. Sie stellen die 5 Funktionen auf und machen eine Fallunterscheidung für y = 0, y = z = 0 und z = 0.

Sie kommen aber nicht auf das selbe wie ich. Lösung P1(\( \frac{1}{\sqrt{2}} \),-\( \frac{1}{\sqrt{2}} \,0)  und  P2( \frac{1}{\sqrt{2}} \),0,-\( \frac{1}{\sqrt{2}} \))


Was mache ich falsch mit der Determinatenmethode?


Liebe Grüsse

VuVanHang

Avatar von

1 Antwort

0 Daumen
 
Beste Antwort

Die Determinante lautet$$D = 6yz(y-z) $$die ist \(=0\), wenn \(y=0\) oder(!) \(z=0\) oder(!) \(y=z\) ist. Diese drei Fälle gilt es zu unterscheiden.

In den ersten beiden Fällen bekommt man jeweils zwei Lösungen, von denen das Maximum für \(f\) immer bei \(f=x=\frac 12 \sqrt 2\) liegt. Der dritte Fall liefert ebenso zwei Lösungen mit \(x_{1,2}\approx \pm0,665\) und \(z_{1,2} \approx \mp 0,528\). Da aber \(+0,665 \lt \frac 12 \sqrt 2\) ist, liegt hier kein (absolutes) Maximum für \(f\) vor.

Avatar von 48 k

ach so, habe demfall bei der determinanten umformung gescheitert, danke

sorry, muss nochmal nachfragen: wie kommst du auf die f(max)= \( \sqrt{2} \)×\( \frac{1}{2} \)?

sorry, muss nochmal nachfragen: wie kommst du auf die f(max)= 2–√×12?

Fall 1: \(y=0\) Einsetzen in die Nebenbedingungen:$$F=x^2+0^2+z^2-1=0 \\ G = x^3+ 0^3 + z^3 = 0$$Bleibt man mit \(x, \, z\) in \(\mathbb{R}\), so folgt aus der zweiten Gleichung \(z=-x\). Das wiederum in die erste Gleichung eingesetzt:$$\begin{aligned}x^2 + (-x)^2 - 1 &= 0 \\ x^2 &= \frac 12 \\ x_{1,2} &= \pm \frac 12 \sqrt 2\end{aligned}$$Und damit die beiden Lösungen $$\begin{aligned}(x,y,z)_1 &= \left( +\frac 12 \sqrt 2, \, 0, - \frac 12 \sqrt 2\right) \\ (x,y,z)_2 &= \left( -\frac 12 \sqrt 2, \, 0, +\frac 12 \sqrt 2\right) \end{aligned}$$von denen die erste den größeren Wert für \(f=x\) liefert.

Fall 2: \(z=0\) .. geht genauso.

Ein anderes Problem?

Stell deine Frage

Willkommen bei der Mathelounge! Stell deine Frage einfach und kostenlos

x
Made by a lovely community